Difference between revisions of "2015 AMC 8 Problems/Problem 24"

m (Problem)
(Solution 1)
Line 1: Line 1:
 
Big balck men
 
Big balck men
 
==Solution 1==
 
On one team they play <math>3N</math> games in their division and <math>4M</math> games in the other.  This gives <math>3N+4M=76</math>.
 
 
Since <math>M>4</math> we start by trying M=5. This doesn't work because <math>56</math> is not divisible by <math>3</math>.
 
 
Next, <math>M=6</math> does not work because <math>52</math> is not divisible by <math>3</math>.
 
 
We try <math>M=7</math> does work by giving <math>N=16</math> ,<math>~M=7</math> and thus <math>3\times 16=\boxed{\textbf{(B)}~48}</math> games in their division.
 
 
<math>M=10</math> seems to work, until we realize this gives <math>N=12</math>, but <math>N>2M</math> so this will not work.
 
  
 
==See Also==
 
==See Also==

Revision as of 17:44, 22 December 2024

Big balck men

See Also

2015 AMC 8 (ProblemsAnswer KeyResources)
Preceded by
Problem 23
Followed by
Problem 25
1 2 3 4 5 6 7 8 9 10 11 12 13 14 15 16 17 18 19 20 21 22 23 24 25
All AJHSME/AMC 8 Problems and Solutions

The problems on this page are copyrighted by the Mathematical Association of America's American Mathematics Competitions. AMC logo.png